Composite form of Boole's Rule

  • Thread starter Thread starter says
  • Start date Start date
  • Tags Tags
    Composite Form
Click For Summary
The discussion focuses on developing a composite form of Boole's rule for integrating a function over a specified interval [a, b]. The proposed formula for the integral involves dividing the interval into 4n segments, where n is a positive integer, leading to a specific weighted sum of function evaluations. Participants express uncertainty about deriving the error bound formula, which is given as - (8h^7/945) * d^6g/dt^6 (ξ) for some ξ in [0, 4h]. The conversation emphasizes the need to understand the implications of increasing the number of intervals on the accuracy of the approximation. Overall, the thread seeks clarity on both the composite formula and the associated error bound.
says
Messages
585
Reaction score
12

Homework Statement


Develop a composite form of Boole's rule for an integral of the form ∫ f(x) dx, where the bounds of integration are from [a,b].

Determine the error bound formula for the composite form of Boole's rule.

∫ g(t) dt = h/45[14g(0)+64g(h)+24g(2h)+64g(3h)+14g(4h)] - (8h7/945)*d6g/dt6 (ξ)

for some ξ ∈ [0,4h]
bounds of integration are [0,4h]

Homework Equations


∫ g(t) dt [bounds of integration [a,b]
a=a
b=a+nh

The Attempt at a Solution


∫ g(t) dt = h/45[14g(0)+64g(h)+24g(2h)+64g(3h)+14g(4h)]

∫ g(t) dt = 2h/45[7g(0)+32g(h)+12g(2h)+32g(3h)+7g(4h)]

∫ g(t) dt = 2h/45[7g(a)+32g(a+h)+12g(a+2h)+32g(a+3h)+7g(a+4h)]

∫ g(t) dt = 2h/45[7g(a)+32g(a+h)+12g(a+2h)+32g(a+3h)+7g(a+4h)]

∫ g(t) dt = 2h/45[7g((a)+(a+4h))+32g((a+h)+(a+3h))+12g(a+2h)]

I think that is the composite of Boole's rule. I'm not sure how to determine the error bound formula for the composite form of Boole's rule though. Any help would be much appreciated :)
 
Physics news on Phys.org
says said:

Homework Statement


Develop a composite form of Boole's rule for an integral of the form ∫ f(x) dx, where the bounds of integration are from [a,b].

Determine the error bound formula for the composite form of Boole's rule.

∫ g(t) dt = h/45[14g(0)+64g(h)+24g(2h)+64g(3h)+14g(4h)] - (8h7/945)*d6g/dt6 (ξ)

for some ξ ∈ [0,4h]
bounds of integration are [0,4h]

Homework Equations


∫ g(t) dt [bounds of integration [a,b]
a=a
b=a+nh

The Attempt at a Solution


∫ g(t) dt = h/45[14g(0)+64g(h)+24g(2h)+64g(3h)+14g(4h)]

∫ g(t) dt = 2h/45[7g(0)+32g(h)+12g(2h)+32g(3h)+7g(4h)]

∫ g(t) dt = 2h/45[7g(a)+32g(a+h)+12g(a+2h)+32g(a+3h)+7g(a+4h)]

∫ g(t) dt = 2h/45[7g(a)+32g(a+h)+12g(a+2h)+32g(a+3h)+7g(a+4h)]

∫ g(t) dt = 2h/45[7g((a)+(a+4h))+32g((a+h)+(a+3h))+12g(a+2h)]

I think that is the composite of Boole's rule. I'm not sure how to determine the error bound formula for the composite form of Boole's rule though. Any help would be much appreciated :)

I think that what the question wants you to find is a formula for ##\int_a^b f(x) \, dx## obtained by splitting the interval ##[a,b]## into ##4n## pieces of length ##h = (b-a)/(4n)## each. So, instead of having just 4 intervals you might have 40 or 400 intervals---some multiple of 4, anyway.
 
Ray Vickson said:
I think that what the question wants you to find is a formula for ∫baf(x)dx∫abf(x)dx\int_a^b f(x) \, dx obtained by splitting the interval [a,b][a,b][a,b] into 4n4n4n pieces of length h=(b−a)/(4n)h=(b−a)/(4n)h = (b-a)/(4n) each. So, instead of having just 4 intervals you might have 40 or 400 intervals---some multiple of 4, anyway.

Ok, so I think I've got a formula that splits the interval up in 4n pieces of length h. I've inserted it below (I=...)

I = (2h/45) [7f(a)+32f(a+h)+12f(a+2h)+32f(a+3h)+7f(a+4h)] + (2h/45) [7f(a+4h)+32f(a+5h)+12f(a+6h)+32f(a+7h)+7f(a+8h)] + ... + (2h/45) [7f(a+(n-4)h)+32f(a+(n-3)h)+12f(a+(n-2)h)+32f(a+(n-1)h)+7f(a+nh)]

I = (2h/45)[7f(a+(n-4)h)+32f(a+(n-3)h)+12f(a+(n-2)h)+32f(a+(n-1)h)+7f(a+nh)]

I'm not sure how I would determine the error bound formula for the composite rule though.

h = (b-a)/(4n)

Error bound formula = - (8h7/945)*d6g/dt6 (ξ)
for some ξ ∈ [0,4h]
 
Question: A clock's minute hand has length 4 and its hour hand has length 3. What is the distance between the tips at the moment when it is increasing most rapidly?(Putnam Exam Question) Answer: Making assumption that both the hands moves at constant angular velocities, the answer is ## \sqrt{7} .## But don't you think this assumption is somewhat doubtful and wrong?

Similar threads

  • · Replies 2 ·
Replies
2
Views
1K
  • · Replies 18 ·
Replies
18
Views
2K
  • · Replies 3 ·
Replies
3
Views
3K
  • · Replies 3 ·
Replies
3
Views
2K
  • · Replies 3 ·
Replies
3
Views
2K
  • · Replies 1 ·
Replies
1
Views
3K
Replies
6
Views
2K
  • · Replies 2 ·
Replies
2
Views
2K
  • · Replies 36 ·
2
Replies
36
Views
5K
  • · Replies 19 ·
Replies
19
Views
3K